跳到主要内容

标记为[静电]的问题

这个标签没有使用指南。

41问题
筛选依据
排序依据
标记为
5 投票
0 答案
148 意见

Galois上升-什么时候变化是Weil限制?

设$L|K$是度$d$的有限Galois扩张,$X$是$K$上的变分。有没有一个简单的标准,类似于伽罗瓦下降,允许确定$X$是否是Weil限制(。。。
Béranger Seguin的用户头像
5 投票
1 回答
244 意见

“广义希尔伯特90”/伽罗瓦下降的逆

希尔伯特90的以下概括可以在塞雷的《洛高兵团》(法国版第十章,§1,ex.2,p.160)中找到,也可参见此问题:定理:如果$L|K$是有限Galois扩张。。。
Béranger Seguin的用户头像
1 投票
0 答案
195 意见

我们能在代数闭包的基改变后检查同构的光滑性吗?

我知道平滑度是基上的fppf局部的,但这还不够,因为取代数闭包并不是有限的。我问这个的原因是因为我想要一个简单/快速的。。。
TCiur的用户头像
4 投票
0 答案
146 意见

伽罗瓦血统的唯一性

设$Y,Z$为$\mathbb{F} (p)$-schemes,它们都是$X$over$\overline{\mathbb方案的模型{F} (p)}$ . 设$F$是$\overline{\mathbb的绝对Frobenius{F} (p)}$,如果$1_Y\times F$和$1_Z\times。。。
傻瓜兔子的用户头像
1 投票
0 答案
170 意见

向量丛截面的Galois下降问题

设$\pi:Y\rightarrowX$是具有Galois群$G$的两个光滑投影簇之间的有限元Galois态射。设$\mathcal{E}$是$X$上的向量束。那么$\pi^*\mathcal{E}$是一个$G$。。。
Hajime_Saito的用户头像
2 投票
0 答案
133 意见

同伦不动点与余代数

参考此答案的最后一部分https://mathoverflow.net/a/225403/170683,我想了解在Galois组$G$中如何覆盖$f\colon X\到Y=X/G$(我猜。。。
Nikio的用户头像
  • 351
投票
0 答案
206 意见

Galois群如何作用于各种Néron–Severi群?

让$K/K$是Galois组$\Gamma$的Galois扩展,让$X$是$K$上的一个变体。假设$X(k)\neq\varnothing$或$\mathrm{Br}(k)=0$,即$k$的Brauer群。Hochschild-。。。
Mobius的用户头像
投票
0 答案
201 意见

Hopf代数的Galois下降

在这里,关于为什么交换Hopf代数的分类要求域是代数闭的问题中,讨论了Galois下降的概念,给出了一个很好的答案。因为我。。。
JamalS的用户头像
2 投票
1 回答
171 意见

关于阿贝尔自同构群对象扭曲的一个简单事实的概念解释

假设$X,X'$是域$k$上的两个对象(例如,亏格1曲线),使得在代数闭包$X{\overlinek}\congX'{\overrinek}$上,并且$Aut_{\overbinek}(X)$是阿贝尔的。...
Asvin的用户头像
  • 第7716页
1 投票
1 回答
589 意见

Fpqc-局部常数当且仅当étale-局部常数?

同样在SE。让$\mathcal{F}$成为$S_\mathrm{fpqc}$上的滑轮。我们说$\mathcal{F}$是(有限生成阿贝尔群的)fpqc-局部常数层,如果存在覆盖$(S_i\to S)的fpqc。。。
Z Wu的用户头像
6 投票
1 回答
1公里 意见

扭转自同构上的伽罗瓦作用是同构的吗?

这可能是一个微不足道的问题,我可能忽略了一些东西:假设$k$是一个代数闭包$\在k$上,绝对Galois群$\Gamma$的字段。让$X,Y$是两个不同的。。。
Asvin的用户头像
  • 7,716
2 投票
0 答案
309 意见

有效下降态射的特征

交换环$A\rightarrowB$的忠实平坦态射是一个有效的下降态射。正则单态也是如此(对吗?)。什么是有效下降形态的特征?...
用户化身
2 投票
1 回答
372 意见

$\overline{k}$格式之间的同构上的Galois作用

我有一个问题,关于赋予伽罗瓦作用的方案之间的态射的某种性质。动机来自于菲尔·托斯特森对这个问题的评论。菲尔写道:“如果地图。。。
user267839的用户头像
  • 5,938
1 投票
0 答案
274 意见

伽罗瓦下降法的应用

我不理解Moonens和van der Geers关于Abelian变量的书(第12页)中刚度引理证明之初所做的假设。这里是:问题:为什么假设$k=\。。。
user267839的用户头像
  • 5,938
7 投票
0 答案
325 意见

非线性伽罗瓦下降

这个问题是关于伽罗瓦理论的。所以让$K/K$成为字段的Galois扩展。让我们假设$K/K$是有限维的,尽管通过……可以使一切在有限情况下工作。。。
雅各布·沃纳的用户头像

15 30 50 每页